ABFM HEALTH COUNSELING AND PREVENTIVE CARE

अब Quizwiz के साथ अपने होमवर्क और परीक्षाओं को एस करें!

You see a 65-year-old female for a health maintenance visit. During substance use screening she notes that she has been consuming alcohol much more frequently in the past 6 months. She says that she occasionally binges and drinks eight or more drinks in one day but that her alcohol consumption has not interfered with her relationships or her job. Her past medical history includes stage 4 chronic kidney disease due to hypertension. She does not have a history of liver disease, and liver function tests performed prior to the visit were within normal limits. Her history is negative for depression and her Patient Health Questionnaire-9 (PHQ-9) score is 5.Your assessment indicates that she has alcohol use disorder of moderate severity. Which one of the following would be an appropriate pharmacologic intervention? Acamprosate Citalopram (Celexa) Disulfiram (Antabuse) Lorazepam (Ativan) Naltrexone

E A systematic review of 53 randomized trials, including 9140 patients, found that oral naltrexone increased abstinence rates (number needed to treat [NNT]=20) and decreased heavy drinking (NNT=12) (SOR A). There is some evidence that disulfiram reduces drinking days (SOR B), but little evidence for promoting abstinence. Adherence is a key predictor of the success of disulfiram. In addition, supervised use of disulfiram seems to have better outcomes.A systematic review generally showed that pharmacologic treatment was more efficacious than placebo in terms of controlling drinking, achieving abstinence, and reducing drinking days, alcohol consumption, and craving. Naltrexone would be indicated in a patient with normal liver enzyme levels, increased cravings, and possible binge drinking.Acamprosate is contraindicated in patients with stage 4 kidney disease (glomerular filtration rate <30 mL/min/1.73 m2) and disulfiram is contraindicated in patients with coronary artery disease. Disulfiram is also contraindicated in patients with a history of psychosis, because it can increase dopamine levels and induce psychotic episodes in these patients.Gabapentin may also be used to treat alcohol withdrawal, and there is some evidence that it can reduce cravings. Lorazepam would not be indicated for long-term therapy. Although there are studies showing some benefit to using SSRIs in patients with concomitant depression and alcohol use disorder, an SSRI would not be indicated in a patient with a low Patient Health Questionnaire-9 (PHQ-9) score.

NEW SCREENING GUIDELINES FOR LOW DOSE CT SCAN

start LDCT at age 50 in adults who have a 20 pack year history.

A 32-year-old pregnant female at 32 weeks gestation presents to your office for prenatal care. Which one of the following is true regarding Tdap vaccine for this patient? She should receive Tdap even if she received it 14 months ago during her most recent pregnancy She should receive Tdap even if she received it at 16 weeks gestation when she sustained a laceration She does not need Tdap if she has received it in the past 3 years She does not need Tdap if she received Td vaccine in the past 12 months

A A Tdap booster is recommended during every pregnancy. It should be given at 27-36 weeks gestation if at all possible. This allows transmission of pertussis antibodies to the fetus before birth and will protect the infant for the first 2-3 months before primary immunizations take effect. If the mother received Tdap early in the pregnancy an additional dose is not required. Td vaccine does not provide protection against pertussis.

A 2-month-old male is brought to your office by his parents for a well child visit. They also have a 3-year-old son who is obese. The mother asks what to do to keep the infant's weight under control over the next few years.Which one of the following recommendations is supported by good evidence? Excluding all juices from the diet until 4 years of age Limiting juice intake to no more than 8 oz per day from 1-6 years of age Restricting chocolate milk intake after 12 months of age Restricting whole milk and using reduced-fat or fat-free milk beginning at 12 months of age

D Pediatric obesity has become epidemic. Measures to prevent this condition should be shared with parents as early as possible. For infants, major recommendations to reduce the risk of obesity include breastfeeding (SOR A), avoiding television and computer screen time (SOR C), avoiding premature introduction of solid foods (SOR C), avoiding high-calorie beverages with low nutritional value (SOR C), and educating parents to be role models of healthy lifestyles (SOR C). Breastfeeding after the age of 12 months has been associated with a 47% reduction in obesity. Cow's milk and fruit juice can be introduced at 12 months of age. Providing 100% fruit juice can be an important component of fruit intake in children who may not be able to access fresh fruits and eliminating them may cause nutritional deficiencies. It is important to only use 100% juice and to not exceed 4-6 oz daily for children 1-6 years of age. When consumed within the Dietary Guidelines for Americans recommendations, 100% fruit juice is not associated with overweight/obesity or childhood dental caries and does not compromise fiber intake. While there have been concerns in the past about low-fat diets and their effect on brain development, low-fat or fat-free milk is appropriate at this age, especially if there are concerns about obesity or a family history of cardiovascular disease (SOR A). Evidence does not support restricting flavored milk products to reduce the prevalence of obesity.

A 24-year-old female sees you for a preconception visit and removal of her IUD. This will be her first pregnancy and she tells you that she has smoked ¼-½ pack of cigarettes a day for the past 5 years.Which one of the following would be appropriate advice regarding the risks from smoking? Smoking during pregnancy increases the risk of attention-deficit/hyperactivity disorder Smoking during pregnancy increases the risk of clubfoot Smoking during pregnancy increases the risk of congenital atrial septal defects Stopping smoking now will reduce the increased risk of orofacial defects in her infant Reducing smoking now will reduce the risk of preterm delivery

D There are many reproductive problems related to smoking, including conception delay and both primary and secondary infertility; an increased risk of ectopic pregnancy and spontaneous abortion; an increased risk of abruption, preterm rupture of membranes, placenta previa, and premature delivery; and increased perinatal morbidity and mortality, including stillbirth, low birth weight, and SIDS-related deaths. The 2001 Surgeon General's Report on women and smoking makes it clear that stopping smoking during pregnancy reduces and sometimes eliminates many of these consequences.Small for gestational age (SGA) infants are a dose-dependent outcome of maternal smoking, with an odds ratio (OR) of 2.11 when women smoke throughout pregnancy. Risks for prematurity (OR 1.15) and fetal death (OR 1.15) are also increased. The risk of having an SGA infant is avoided if smoking is reduced, but the risks for prematurity and increased fetal death are not.In 2014 the U.S. Surgeon General issued a new report on the health consequences of smoking that noted that the evidence was strong enough to infer a causal link between maternal smoking and orofacial clefts. This was still true when the Surgeon General issued a report on smoking cessation in 2020. No link could be inferred, however, between smoking and other congenital defects, including clubfoot, gastroschisis, and atrial septal defects. There is no evidence that maternal smoking leads to increased rates of childhood attention-deficit/hyperactivity disorder.

According to the U.S. Preventive Services Task Force, which one of the following should be offered BRCA-mutation genetic counseling, based on the information provided? A 35-year-old female whose 72-year-old grandfather was recently diagnosed with breast cancer A 48-year-old female whose 58-year-old sister was recently diagnosed with ovarian cancer A 50-year-old female with an elevated Gail Model risk score whose mother was diagnosed with breast cancer at age 70 and whose sister was diagnosed with breast cancer at age 62 A 55-year-old female whose mother was diagnosed with breast cancer at age 61

A A history of breast cancer in any male relative justifies referral for BRCA testing (B recommendation). According to the U.S. Preventive Services Task Force (USPSTF), patients should generally NOT be referred for BRCA counseling or screening because of breast cancer in a female first degree relative unless the diagnosis was made before the age of 55 (B recommendation). One exception is Ashkenazi Jewish women, who are at increased risk for BRCA mutations and should be considered for testing if there is a family history of breast cancer in one first degree relative (B recommendation). Bilateral breast cancer in a first degree relative also justifies referral for BRCA testing (B recommendation).BRCA mutations increase the risk for both breast and ovarian cancer before age 70 (35%-84% and 10%-50%, respectively). A family history of both types of cancer in first or second degree relatives significantly increases the risk of having a BRCA mutation, and screening is recommended (B recommendation).Validated tools evaluated by the USPSTF include the Ontario Family History Assessment Tool, Manchester Scoring System, Referral Screening Tool, Pedigree Assessment Tool, 7-Question Family History Screening Tool, International Breast Cancer Intervention Study instrument (Tyrer-Cuzick), and brief versions of BRCAPRO. General breast cancer risk assessment models, such as the National Cancer Institute Breast Cancer Risk Assessment Tool (Gail Model), are not designed to identify BRCA-related cancer risk and should not be used for this purpose.Patients should be offered genetic counseling to thoroughly discuss ramifications of testing before ordering BRCA testing.

A 24-year-old female in the second trimester of her first pregnancy is concerned that she may contract influenza and endanger her baby's health. Her due date is in December and she plans to breastfeed. She has not received influenza vaccine in the past because she develops hives if she eats eggs.Which one of the following would be an appropriate recommendation? She can safely receive trivalent inactivated influenza (TIV) vaccine prior to the upcoming influenza season She can safely receive live attenuated influenza vaccine (LAIV) prior to the upcoming influenza season She should not receive the vaccine this year due to her history of an allergic reaction to eggs She should not take oseltamivir (Tamiflu) for prophylaxis if she is exposed to influenza prior to delivery

A The American College of Obstetricians and Gynecologists, the American Academy of Family Physicians, and the CDC recommend influenza vaccine for all women who will be pregnant during influenza season. It has been shown to reduce the risk of influenza-associated acute respiratory infection in pregnant women by about one-half and reduces a pregnant woman's risk of being hospitalized with influenza by an average of 40%. The CDC recommends use of injectable influenza vaccines, including inactivated influenza vaccines and recombinant influenza vaccines. The nasal spray vaccine, which is a live attenuated influenza vaccine, is not recommended during pregnancy. Influenza vaccine is also recommended for women who are breastfeeding (SOR A). Current CDC guidelines recommend oseltamivir as the preferred treatment for pregnant women with suspected influenza.People with egg allergies such as hives, but no previous reaction to influenza vaccine in the past, no longer need to be observed for an allergic reaction for 30 minutes after receiving influenza vaccine. Monitoring for 30 minutes may be done if this is the first time the patient has received influenza vaccine but is not mandatory according to the CDC. Any licensed and recommended influenza vaccine that is otherwise appropriate for the recipient's age and health status may be used.Patients who report reactions to egg involving symptoms other than hives, such as angioedema, respiratory distress, lightheadedness, or recurrent emesis, or who required epinephrine or another emergency medical intervention, may also receive any licensed and recommended influenza vaccine that is otherwise appropriate for the recipient's age and health status. If an egg-based vaccine is used, it should be administered in an inpatient or outpatient medical setting, including, but not necessarily limited to hospitals, clinics, health departments, and physician offices. Vaccine administration should be supervised by a health care provider who is able to recognize and manage severe allergic conditions. These precautions are not necessary if the recombinant or cell culture-based vaccines are used.

A 67-year-old female with a history of coronary artery disease tells you that a friend of hers has recommended taking 400-800 IU of vitamin E daily.Which one of the following would you tell her regarding this supplement? It is effective for delaying progression of macular degeneration It has been shown to reduce the risk of Alzheimer's dementia It may reduce the risk of myocardial infarction in the general population Supplementation of 400-800 IU daily would be harmful

A Vitamin E supplementation appears to have a beneficial effect on progression of macular degeneration. It has not been proven to prevent Alzheimer's dementia or mild cognitive impairment. Although vitamin E supplementation has not shown any benefit for prevention of cardiovascular disease in the general population, clinical trial data from the HOPE (Heart Outcomes Prevention Evaluation), ICARE (Israel Cardiovascular Events Reduction with Vitamin E), and Women's Health Study clinical trial data has shown that in individuals with diabetes mellitus and the Hp2-2 genotype, vitamin E supplementation is associated with an approximately 35% reduction in cardiovascular disease. A meta-analysis from 2014 showed that vitamin E is safe in dosages up to 800 IU daily.

A 53-year-old male asks your advice about the use of supplements to prevent cancer and cardiovascular disease. Which one of the following supplements should he AVOID? β-Carotene Multivitamins with folic acid Vitamin A Vitamin C

A β-Carotene has been found in clinical trials with smokers to be related to increased rates of lung cancer and overall mortality. Furthermore, in 2003 and again in 2014 the U.S. Preventive Services Task Force specifically recommended against the use of β-carotene for chemoprevention of cancer for all adults (D recommendation). The evidence for vitamins A and C, and for multivitamins with folic acid, is insufficient to recommend for or against their use for prevention of cancer or cardiovascular disease (I recommendation).

You see a 47-year-old male for follow-up of elevated blood pressure. He does not have kidney disease or diabetes mellitus. His family history is notable for hypertension in his mother and older brother. He does not exercise regularly and says he eats a "normal diet." He does not drink alcohol during the week, but says he has 3-4 12-oz cans of beer each day on the weekend. On examination he has a blood pressure of 147/86 mm Hg and a BMI of 32 kg/m2.The patient wants to avoid medications at this time. Which one of the following interventions would be most effective for reducing his blood pressure? Reducing dietary sodium intake to ≤2400 mg daily Following the Dietary Approaches to Hypertension (DASH) diet Reducing alcohol consumption to ≤2 drinks daily on the weekends Losing 5 kg (11 lb) of body weight Engaging in 30 minutes of aerobic physical activity on most days of the week

B According to the 2017 American College of Cardiology/American Heart Association hypertension guidelines, limiting daily alcohol intake to 1-2 drinks in men and 1 drink in women has the least impact on lowering systolic blood pressure (SBP) among nonpharmacologic interventions in hypertensive individuals, with an anticipated reduction of 4 mm Hg in SBP. Adoption of the Dietary Approaches to Hypertension (DASH) diet is the most effective dietary intervention for reducing SBP, with an approximate average reduction of 11 mm Hg. The DASH diet is rich in fruits, vegetables, whole grains, and low-fat dairy products, with reduced amounts of saturated and total fat compared to the typical American diet. Weight loss is a core recommendation in overweight and obese individuals, with an expected drop in SBP of about 1 mm Hg per kg of weight loss. Engaging in regular aerobic exercise 90-150 minutes per week is also an effective intervention, leading to expected reductions of 5-8 mm Hg in SBP. Reduction of dietary sodium intake by at least 1000 mg daily (with an optimal goal of <1500 mg daily) would be expected to reduce SBP by 5-6 mm Hg.

A 42-year-old female sees you for a routine well woman visit. She mentions that she has started dating after her recent divorce and asks for advice about using condoms to prevent pregnancy and decrease her risk for sexually transmitted infections (STIs).Which one of the following would be accurate advice? Natural membrane condoms (also called "natural" or "lambskin" condoms) are recommended for STI prevention Failure of the condom to prevent STI transmission or unintended pregnancy is usually due to breakage Condom use may decrease the risk of herpes simplex virus type 2 transmission from an infected female partner to an uninfected male partner Condoms are regulated by the FDA as a medical device and are subject to random testing and sampling for quality

D Family physicians can play an important role in the prevention and treatment of sexually transmitted infections (STIs). The focus of primary STI prevention should be on helping patients change sexual behaviors that put them at risk for infection. Physicians should regularly obtain adequate sexual histories from their patients and address risk reduction.STI counseling should be client centered, meaning it should include risk reduction messages that are specific to and relevant to the individual patient, as well as education on the specific actions that can reduce risk. Condoms are a common choice for reducing STI risk, as well as preventing pregnancy. The physician can help ensure that the patient is educated about the selection and use of condoms.There are two types of non-latex condoms on the market. One is made of polyurethane or another synthetic material, and is equivalent to latex condoms in efficacy for prevention of STI transmission and pregnancy. These can be used by people who have latex allergies. Natural membrane condoms (also called lambskin condoms) are made from the cecum of lambs and have pores or openings in the material up to 1500 nm in diameter. This represents 10 times the diameter of the human immunodeficiency virus (HIV) and 25 times the diameter of the hepatitis B virus. Thus, natural material condoms are NOT recommended for the prevention of STD transmission. However, this type of condom has an efficacy similar to that of latex for the prevention of pregnancy.Limited evidence shows that correct and consistent use of the male condom may decrease the risk of transmission of the herpes simplex virus type 2 (HSV-2) from an infected male to his uninfected female partner. However, subgroup analysis revealed no significant difference in transmission of HSV-2 from an infected woman to her uninfected male partner if a condom was used. There are no systematic reviews or randomized, controlled trials on female condoms and HSV-2 transmission. Male latex condoms are also effective in preventing transmission of HIV and decrease transmission of Chlamydia, gonorrhea, and trichomoniasis. They may also reduce the risk that a woman will develop pelvic inflammatory disease, and they may reduce the risk of HPV-associated disease.Inconsistent or incorrect application of the condom is by far the most common reason for failure to prevent STI transmission or pregnancy. In the United States, the average rate of breakage during sexual intercourse and withdrawal is only 2/100. Condoms are subject to random sampling and testing by the FDA, and each latex condom manufactured in this country is tested electronically by the manufacturer for holes.

A 29-year-old female who recently immigrated to the United States from Kenya presents for a routine well woman visit. She has been in a monogamous relationship for the past 3 years and plans to get married in 6 months. She reports only one other sexual partner in her lifetime, with first intercourse at age 23. She has no past history of sexually transmitted infections. She has had Papanicolaou (Pap) tests according to the recommended schedule, and all have been normal. She had a levonorgestrel IUD (Mirena) placed 2 years ago. She has not received HPV vaccine and is interested in doing what she can to reduce her risk of HPV.Which one of the following would be the most appropriate advice regarding HPV vaccine? It is not indicated for her because there is no significant benefit for those over age 26 It is not FDA approved for women over age 26 She should receive the vaccine at this visit and a second dose in 6 months She should receive the vaccine at this visit, a second dose in 1-2 months, and a third dose in 6 months

D According to the CDC's Advisory Committee on Immunization Practices, HPV vaccination is recommended for all women <26 years of age, regardless of risk (SOR A). Ideally, the full vaccination series should be administered before potential exposure to HPV through sexual activity. Women who are sexually active should still be vaccinated, however, including those with a history of genital warts, abnormal Papanicolaou (Pap) smears, or positive HPV DNA tests.If the first dose of vaccine is given before the patient's 15th birthday a two-dose schedule should be followed, with the second dose given 6-12 months after the first dose. The minimum interval is 5 months between the first and second dose. If the second dose is administered after a shorter interval, a third dose should be administered a minimum of 5 months after the first dose and a minimum of 12 weeks after the second dose.A series of three doses is recommended if the vaccine is initiated on or after the patient's 15th birthday. The second dose should be administered 1-2 months after the first dose, and the third dose should be administered 6 months after the first dose.Vaccination is not routinely recommended for those over the age of 26. However, some adults age 27-45 who did not get adequately vaccinated when they were younger may decide to get the HPV vaccine after a discussion with their physician. HPV vaccination of people in this age range provides less benefit, as more of them have been already exposed to HPV. The FDA has approved a supplemental application for a recombinant 9-valent HPV vaccine and expanded the approved use of the vaccine to include women and men age 27-45.

An 18-year-old male presents with a cough and shortness of breath. He started smoking at age 15. He is currently trying to stop smoking by using electronic cigarettes (vaping).You counsel him that vaping increases success rates in smoking cessation for adolescents gradually lowering the dose of nicotine in vaping products helps attenuate withdrawal effects vaping causes less second-hand nicotine exposure than regular cigarettes lung injury can occur even with occasional vaping use

D Current evidence does not suggest that the use of electronic cigarettes (e-cigarettes, or vaping products) for tobacco cessation is effective in adolescents. Studies in adults have shown mixed results. In a meta-analysis of 29 articles, e-cigarettes led to modest cessation rates, with benefits including behavioral and sensory gratification. In other studies participants continued to use e-cigarettes to maintain their habit instead of quitting. A total of 22 toxic substances apart from nicotine were reported in the liquid used in vaping cartridges and in their emissions. Vitamin E acetate, an additive in some vaping products, especially those containing tetrahydrocannabinol, has been strongly linked to the development of e-cigarette or vaping product use-associated lung injury (EVALI), which causes cough, dyspnea, and often gastrointestinal symptoms. EVALI may cause severe alveolar damage, and individuals as young as 15 have died from it. Patients who vape should be asked about all substances they add to vaping products and be counseled to stop.The evidence is insufficient to support the claim that vaping causes less second-hand exposure than regular cigarettes. One study showed that passive exposure to e-cigarette aerosol increased serum levels of cotinine similar to those associated with passive exposure to conventional cigarette smoke. There is no evidence that gradually lowering the nicotine dose in vaping products attenuates nicotine withdrawal.

According to the U.S. Preventive Services Task Force, screening for Chlamydia infection would NOT be recommended for which one of the following patients? A sexually active 17-year-old female who has one male sexual partner and uses condoms, and has never been screened A sexually active 24-year-old female who has had one lifetime male sexual partner, no previous history of sexually transmitted infections, and no symptoms, and was screened 1 year ago A 24-year-old pregnant female at 12 weeks gestation who has had three lifetime sexual partners A sexually active heterosexual 25-year-old male who has had three lifetime female sexual partners

D The U.S. Preventive Services Task Force (USPSTF) recommends screening all sexually active nonpregnant women age 24 or younger, as well as older women who are at increased risk for chlamydial infection, regardless of age (B recommendation). Screening for chlamydial infection is also recommended for all pregnant women regardless of age, and positive results should be followed up by a test-of-cure culture in 3-4 weeks. Routine screening for chlamydial infection is not recommended in nonpregnant women age 25 and older (C recommendation). The USPSTF found that there is currently insufficient evidence to support screening for chlamydial infection in heterosexual men with no risk factors. However, the CDC recommends screening for men who do have risk factors, such as men who have sex with men, men who currently reside in a correctional facility, or men who have been treated at STD clinics.

At a health maintenance visit, a 29-year-old male who only has sex with men tells you that he ended a long-term relationship about 1 year ago and that he did not use condoms consistently in that relationship because both he and his partner were monogamous at the time. After a one-time sexual encounter 4 months ago he was diagnosed with a Chlamydia infection and treated appropriately. His renal function, HIV, hepatitis B, and hepatitis C tests were negative at that time. He is now dating a new partner but has not been sexually active with that person.In addition to recommending condom use, which one of the following would be appropriate at this time? Retesting for Chlamydia Retesting for hepatitis C A herpes simplex 2 antibody titer Retesting for HIV and starting pre-exposure prophylaxis (PrEP)

D The U.S. Preventive Services Task Force (USPSTF) recommends initiation of pre-exposure prophylaxis (PrEP) for HIV in the following populations: Men who have sex with men, are sexually active, and are in a sexual relationship with a partner who is HIV positive, OR who use condoms inconsistently during receptive or insertive anal sex, OR who have had syphilis, gonorrhea, or a Chlamydia infection within the past 6 months Heterosexually active women and men who are in a sexual relationship with a partner who is HIV positive, OR who use condoms inconsistently during sex with a partner whose HIV status is unknown and who is at high risk, such as a person who injects drugs or a man who has sex with men and women, OR who have had syphilis or gonorrhea within the past 6 months Persons who inject drugs and share drug-injection equipment, OR who are at risk for sexual acquisition of HIV (see above) Two medications have been approved for use as PrEP by the FDA: emtricitabine/tenofovir disoproxil, 200 mg/300 mg once daily, and emtricitabine/tenofovir alafenamide, 200 mg/25 mg once daily.Studies have shown that daily PrEP reduces the risk of HIV from sex by about 99%. In people who inject drugs, daily PrEP reduces the risk of HIV by at least 74%. PrEP is much less effective if it is not taken consistently.Recommending abstinence alone is usually not sufficient. Using condoms 100% of the time will reduce transmission of HIV by 80%, and will also reduce transmission of other sexually transmitted infections (STIs). Since this patient had an STI 4 months ago, PrEP therapy is indicated. Testing for herpes-virus antibodies is not indicated and another test of cure for Chlamydia infection is not needed. Hepatitis B status should be determined before initiation of PrEP, but this patient has recently been tested so testing is not necessary at this time. It is not necessary to determine the patient's hepatitis C status before starting PrEP, and given the patient's recent test it would not be indicated for other reasons at this time.

A male who was born in 1970 comes to your office for a preoperative examination for an orthopedic procedure on his knee. He is otherwise healthy and does not take any medications, but he has not seen a physician for 6 years. He used illicit drugs for a brief period at age 23 but has not done so since that time and has had three sexual partners, all of them female. You use this opportunity to counsel him on preventive health screenings, including hepatitis C.Which one of the following is true regarding screening for hepatitis C? The high cost of treatment outweighs the potential benefit of screening The CDC recommends testing for hepatitis C virus every 3-5 years in patients who have a history of drug injection The U.S. Preventive Services Task Force recommends routine screening for hepatitis C only for those born between 1945 and 1965 This patient should be screened with hepatitis C RNA polymerase chain reaction (PCR) testing Screening all adults up to age 80 regardless of risk profile is recommended

E In 2019 the U.S. Preventive Services Task Force (USPSTF) recommended screening all patients 18-79 years of age at least once for hepatitis C with the anti-HCV antibody test. Detection of hepatitis C virus (HCV) RNA by polymerase chain reaction (PCR) testing provides evidence of active HCV infection, confirms the diagnosis, and is used in monitoring the antiviral response to therapy. Quantitative PCR is used to determine viral load. The CDC previously recommended screening for people born between 1945 and 1965, but that has been expanded.HCV is the most common chronic bloodborne pathogen in the United States and a leading cause of complications from chronic liver disease. Before the COVID pandemic, HCV infection was associated with more deaths than the top 60 other reportable infectious diseases combined, including HIV. The most important risk factor for HCV infection is past or current injection drug use. In the United States an estimated 4.1 million people have past or current HCV infection, based on a positive test for the anti-HCV antibody. Approximately 2.4 million persons with a positive antibody test have a current infection based on results of molecular assays for HCV RNA and would be potential candidates for treatment. Treatment results in very high levels of virus remission.Cases of acute HCV infection increased approximately 3.5-fold between 2010 and 2017. The increased incidence has mostly affected young white people who inject drugs, especially those living in rural areas. There has also been an increase in the number of women age 15-44 years with HCV infection. There is no recommended testing frequency for high-risk individuals at this time.

Moderate daily alcohol use has generally been defined as two or fewer standard alcoholic drinks per day for men and one or fewer drinks per day for women. Which one of the following would be considered a standard alcoholic drink? 1.5 oz of 80-proof brandy 2 oz of 80-proof whiskey 8 oz of red wine 12 oz of malt liquor 16 oz of light beer

A According to the National Institute of Alcohol Abuse and Alcoholism, a standard alcoholic drink is equivalent to 12 oz of beer; 8.5 oz of malt liquor; 5 oz of table wine; 3.5 oz of fortified or dessert wine; 2.5 oz of cordial, liqueur, or aperitif; and 1.5 oz of spirits (one jigger of gin, vodka, whiskey, etc.) (SOR C).

A 72-year-old male sees you for the first time to establish care. His current medications include lisinopril/hydrochlorothiazide (Zestoretic) for hypertension, as well as aspirin, 81 mg daily. He says that he feels well but is concerned about prostate cancer because his father died of prostate cancer at the age of 72.The patient's PSA level was normal 4 years ago. He has nocturia one time per night but has not noticed any change in urination lately. His blood pressure is normal today.Which one of the following is supported by the best evidence with regard to prostate cancer screening for this patient? Do not screen for prostate cancer with any examination or laboratory testing Perform a rectal examination, and order a PSA level only if the prostate is enlarged Order a PSA level now and do not perform a rectal examination Order a PSA level now and perform a rectal examination

A Advanced age is the strongest risk factor for the development of prostate cancer. Prostate cancer is found most often in men over the age of 50, with 80% of those with prostate cancer being over age 65. In the United States, Black men have a significantly higher incidence of prostate cancer than white men (217.5 versus 134.5 cases per 100,000 men), and more than twice the prostate cancer mortality rate (56.1 versus 23.4 deaths per 100,000 men). The National Cancer Institute Surveillance, Epidemiology, and End Results (SEER) program has also reported a link between high dietary fat intake and the development of prostate cancer. Men with a family history of prostate cancer are at increased risk for prostate cancer, and for death from the disease.The U.S. Preventive Services Task Force (USPSTF) updated its recommendation in 2018 and now recommends that for men age 55-69 the decision to undergo periodic prostate-specific antigen (PSA) screening for prostate cancer should be an individual one (C recommendation). They recommend against PSA screening for men age 70 and older (D recommendation). The USPSTF concluded that PSA screening may reduce prostate cancer mortality risk, but it is also associated with false-positive results, biopsy complications, and overdiagnosis. In addition, the USPSTF concluded that compared with conservative approaches, active treatments for prostate cancer detected by screening have unclear effects on long-term survival and are associated with sexual and urinary difficulties.In a study of men with normal PSA levels, a digital rectal examination identified an additional 2% of men who were found to have clinical prostate cancer based on further analysis. However, there is very little evidence that screening rectal examinations in asymptomatic men above the age of 70 leads to a reduction in prostate cancer deaths.

A 55-year-old female with newly diagnosed stage 1 hypertension asks you about foods that might help her lower her blood pressure. Foods associated with a reduction in blood pressure include which one of the following? Dark chocolate Milk chocolate Licorice Lean animal protein Cheese

A Daily intake of at least 100 g of polyphenol-rich dark chocolate has been found to be associated with a 3-mm reduction in systolic blood pressure and improved formation of vasodilative nitric oxide. In addition, another study found a statistically significant reduction in blood pressure with the consumption of 40 g of soybean protein. Licorice has been associated with hypertension in humans. The Dietary Approaches to Stop Hypertension (DASH) diet, which has been shown to reduce blood pressure, emphasizes fruits, vegetables, and low-fat dairy products, and a reduced intake of dietary cholesterol, saturated and total fat, sweets, and sugar-containing beverages. Cheese generally has a high amount of sodium. For example, a relatively small (30 g) serving of American cheese contains approximately 400 mg of sodium and the same amount of cheddar cheese contains approximately 200 mg of sodium.

A 3-year-old male is brought to your office by his parents for a well child examination. His family recently moved to the area. His father says that both he and his wife have always had problems controlling their weight, and he asks if their son is likely to have the same problem as he gets older.Factors associated with obesity in adulthood include which one of the following? A birth weight categorized as low for gestational age A high socioeconomic level Being a member of an ethnic minority An increase in BMI of 2 kg/m2 in 1 year

A Risk factors for overweight and obesity include low or high birth weights, low socioeconomic levels, poor eating habits, a change in BMI >3-4 kg/m2 per year, depression, >2 hours per day of sedentary activity such as watching television or playing computer games, and minority status (SOR C).Numerous sources have suggested that prevention of obesity is the best way to combat this epidemic. The U.S. Preventive Services Task Force recommends screening for obesity starting at age 6, with referral to intensive therapy (more than 26 contact hours) for children with a BMI greater than the 95th percentile. The BMI value by itself is not as meaningful when assessing children because normal values are based on age. For example, a 10-year-old male with a BMI of 23 kg/m2 would be considered obese, whereas a 15-year-old male with the same BMI would not. A BMI above the 85th percentile is considered overweight.The American Academy of Pediatrics recommends screening for obesity starting at age 2, with interventions for any child with a BMI between the 85th and 94th percentile. Being obese in childhood increases the risk for obesity in adulthood. Thus, it is currently recommended that physicians screen children annually for risk factors for being overweight.

A 66-year-old female sees you for the first time for a health maintenance visit. She has read recently that there is a new shingles vaccine that was better than the previous version, and she asks whether she should get the new vaccine even though she received the older vaccine at age 60.Which one of the following statements would be accurate advice about the recombinant herpes zoster vaccine (Shingrix)? It is more than 90% effective for preventing herpes zoster and postherpetic neuralgia It contains live attenuated varicella virus and should not be given to patients who are immunodeficient It should not be given to patients who have had varicella zoster in the previous 3 months She should wait at least another 4 years to get the recombinant vaccine If a patient has no recollection of varicella in their lifetime then testing for varicella antibodies is recommended before giving the vaccine

A The recombinant herpes zoster vaccine (Shingrix) is a recombinant subunit vaccine that combines a lyophilized varicella zoster virus glycoprotein E (gE) antigen and an adjuvant system. It does not contain a live virus and is administered in two doses of 0.5 mL each, with the second dose given 2-6 months after the initial dose. It has been shown to be more than 90% effective in preventing herpes zoster and postherpetic neuralgia. Its protective effect remains at 85% for at least 4 years after administration, unlike the previous herpes zoster vaccine (Zostavax), which contains a live attenuated varicella virus and only reduces the risk of herpes zoster by 51% and the risk of postherpetic neuralgia by 67%.The recombinant vaccine is indicated for people over the age of 50, whereas the live attenuated vaccine was recommended for people over the age of 60. The recombinant vaccine is recommended for patients who have had shingles in the past, as well as those who received the live attenuated vaccine in the past. When the older vaccine was available the CDC recommended that patients wait at least 8 weeks after receiving the live attenuated vaccine before getting the recombinant vaccine.There is no specific amount of time a patient who has had herpes zoster must wait before receiving the recombinant vaccine, but it should not be administered during an acute episode of zoster. When vaccinating adults age 50 years and older, there is no need to screen for a history of varicella infection or to conduct laboratory testing for serologic evidence of prior varicella infection. Testing is often a barrier to herpes zoster vaccination, and false negatives are common. Worldwide, more than 99% of adults age 50 years and older have been exposed to varicella zoster virus.

A 52-year-old female sees you for a routine health maintenance visit. On examination her blood pressure is 150/85 mm Hg and her BMI is 34 kg/m2. Laboratory findings include an LDL-cholesterol level of 190 mg/dL, a fasting glucose level of 106 mg/dL, and a 2-hour blood glucose level of 160 mg/dL on an oral glucose tolerance test.When counseling this patient regarding weight loss, which one of the following would be accurate advice? Losing 10 kg (22 lb) could eliminate her need for antihypertensive medications Losing 10 kg would lower her LDL-cholesterol level by about 30 mg/dL Reducing her body weight by 5%-10% would not significantly reduce her risk of developing type 2 diabetes Exercise by itself is not an effective way to prevent type 2 diabetes

A There is substantial evidence that a weight loss of 10 kg (22 lb) confers numerous health benefits, including a reduction of up to 20 mm Hg in systolic blood pressure and a concomitant reduction in the need for antihypertensive medication (SOR A). LDL-cholesterol levels have been estimated to decrease by 1% for every kg lost (SOR C).In a Finnish study of 522 middle-aged men with impaired glucose tolerance, intensive individualized instruction on weight reduction, food intake, and increasing physical activity resulted in a 58% reduction in the incidence of diabetes mellitus compared to the control group. In this study, weight reduction of as little as 5%-10% was found to lower the risk of developing diabetes. In the Diabetes Prevention Program the number needed to treat to prevent one case of incident diabetes was 6.9 for the intensive lifestyle intervention compared to 13.9 for metformin, although the efficacy of metformin approached that of the intensive lifestyle intervention in younger subjects and in those with a higher BMI or fasting plasma glucose level at baseline. The median delay in diabetes onset was estimated at 11 years for the intensive lifestyle intervention and 3 years for metformin.The American College of Sports Medicine and the American Diabetes Association Position paper on prevention of diabetes notes a meta-analysis of 10 cohort studies that found a risk reduction for type 2 diabetes of 0.70 (0.58-0.84) for walking on a regular basis (typically briskly for ≥2.5 hours per week).

A 73-year-old male with diabetes mellitus and hypertension takes metformin (Glucophage), atorvastatin (Lipitor), glipizide (Glucotrol), and lisinopril (Prinivil, Zestril). He is quite active and exercises nearly every day. He has taken aspirin, 81 mg daily, in the past, but it seemed to upset his stomach so he quit taking it about 3 years ago. A recent hemoglobin A1c was 6.9%. He has a BMI of 24 kg/m2. He asks if he should restart low-dose aspirin because his father died at age 80 of a stroke.You should advise the patient that he should take aspirin because the benefits outweigh the risks he should not take aspirin for prevention of cardiovascular disease he would have to take aspirin for just 3 years to realize a benefit because of his BMI he is not at increased risk of adverse outcomes from taking aspirin

B Aspirin use may result in small to moderate harms, including gastrointestinal bleeding, which is more common in men, and hemorrhagic stroke, which is more common in those with a low BMI and Asian ethnic groups. Decisions about the use of low-dose aspirin therapy in primary prevention should take into account the patient's overall risk for cardiovascular disease (CVD), colorectal cancer, and gastrointestinal bleeding.The 2019 American College of Cardiology/American Heart Association guideline for the primary prevention of cardiovascular disease recommends aspirin for adults age 40-70 who have an increased risk for CVD and no significant bleeding risk. The guideline also states that the use of aspirin for primary prevention in adults >70 years of age is potentially harmful. In the patient described in this scenario aspirin may increase the risk of bleeding.In 2016 the U.S. Preventive Services Task Force (USPSTF) changed its former A recommendation rating to a B recommendation for initiating low-dose aspirin use for the primary prevention of CVD and colorectal cancer in adults age 50-59 years who have a 10% or greater 10-year CVD risk, are not at increased risk for bleeding, have a life expectancy of at least 10 years, and are willing to take low-dose aspirin daily for at least 10 years. The USPSTF made a C recommendation for adults age 60-69 years with a 10% or greater 10-year CVD risk. The recommendation states that the decision to initiate low-dose aspirin use should be an individual one. Patients who are not at increased risk for bleeding, have a life expectancy of at least 10 years, and are willing to take low-dose aspirin daily for at least 10 years are more likely to benefit. Those who place a higher value on the potential benefits than the potential harms may choose to initiate low-dose aspirin.

You see a 78-year-old male for an annual Medicare wellness examination. Which one of the following statements is true regarding screening for cognitive impairment? Screening for cognitive impairment is recommended by the U.S. Preventive Services Task Force Screening instruments such as the Mini-Mental State Examination (MMSE) and Montreal Cognitive Assessment (MoCA) have good sensitivity for dementia Screening instruments such as the MMSE and MoCA have good specificity for most neurocognitive disorders The MMSE is a more sensitive test than the MoCA for mild cognitive impairment

B Available screening tests for dementia, such as the Mini-Mental State Examination (MMSE), the Functional Activities Questionnaire (FAQ), the Montreal Cognitive Assessment (MoCA), and others, have good sensitivity but only fair specificity for diagnosing dementia and other neurocognitive disorders. The positive and negative predictive value of these instruments will vary depending on the practice setting and the prevalence of dementia in the patient population (SOR A). In populations with a high prevalence of dementia, such as patients over the age of 85, positive predictive values can be >50%. In populations with a lower prevalence, such as unselected patients age 65-74, the positive predictive value may be closer to 20%. The sensitivity and specificity of these tools is typically lower for mild cognitive impairment than for dementia. The MoCA may be a more sensitive screening tool than the MMSE for mild cognitive impairment.In 2011 Medicare began covering screening for cognitive impairment as part of the annual wellness visit. In 2018 the American Academy of Neurology published recommendations to perform screening with a validated instrument and not rely on subjective assessment of memory to diagnose mild cognitive impairment. If patients are diagnosed with mild cognitive impairment a search for reversible causes should be undertaken because a subset of patients will show improvement in function over time with appropriate intervention and with exercise. Patients should be assessed for functional impairment before being given a diagnosis of dementia.The U.S. Preventive Services Task Force has concluded that the current evidence is insufficient to assess the balance of benefits and harms of screening for cognitive impairment in older adults.

A 29-year-old female says that she generally has 1-2 glasses of wine with dinner on weeknights and up to 3-4 glasses on weekends when she and her husband go out with friends. Your evaluation reveals her to be in excellent physical health with no problems at work or with her social life.According to criteria established by the National Institute on Alcohol Abuse and Alcoholism, her alcohol use would be classified as moderate drinking heavy alcohol use harmful drinking alcohol abuse alcohol dependence

B Categories of alcohol use have been established by the National Institute on Alcohol Abuse and Alcoholism.Moderate drinking is defined as up to one standard drink per day in women and up to two per day in men.Heavy alcohol use includes high-volume drinking, defined as 14 or more standard drinks per week on average for males and 7 or more standard drinks for females, and high-quantity consumption, defined as consumption on any given day of 4 or more standard drinks for males and 3 or more standard drinks for females. Although patients with a heavy alcohol use pattern by definition may not have developed adverse consequences, this pattern places them at risk for such consequences.Alcohol abuse is a maladaptive pattern of drinking leading to clinically significant impairment or distress. Alcohol dependence is characterized by multiple symptoms, including tolerance, signs of withdrawal, and diminished control over drinking.

Which one of the following would be appropriate for the patient described? Annual MRI of the breast for a 65-year-old breast cancer survivor, beginning 6 months after completing breast cancer treatment Annual screening mammography for a 25-year-old female Hodgkin's disease survivor who was treated with chest irradiation Follow-up carcinoembryonic antigen testing every 3 months for 1 year and then every 6 months for 4 more years for a 65-year-old male colon cancer survivor A digital rectal examination and monitoring of PSA levels every 6 months for a 72-year-old prostate cancer survivor

B Female Hodgkin's disease survivors treated with chest irradiation are at increased risk of developing breast cancer and surveillance should be started at 25 years of age (SOR C). Breast cancer patients should be counseled that intensive surveillance using laboratory and imaging tests does not improve overall survival or quality of life. However, monthly breast self-examination, annual mammography of preserved breast tissue, and a careful history and physical examination every 6 months for 5 years are recommended (SOR C). A Cochrane review, based on two randomized, controlled trials, found that less-intensive follow-up strategies based on periodic clinical examinations and annual mammography seem as effective as more-intense surveillance. Any positive findings on the history and physical examination would certainly warrant further investigation, however.Although expert opinion recommends frequent carcinoembryonic antigen (CEA) testing and CT scans for follow-up of colorectal cancer patients, there is insufficient evidence to support any optimal combination of tests or frequency of clinical follow-up. CEA testing and CT scans every 6 months versus at 1 and 3 years did not yield a clinically significant difference in 5-year mortality or cancer-specific mortality in a 2018 meta-analysis of patients with stage 2 or 3 colorectal cancer. Another meta-analysis from 2015 demonstrated that more-intensive follow-up was not associated with an improvement in cancer-specific survival nor with an increased detection of total tumor recurrences. A 2016 Cochrane review on this subject found no overall survival benefit for increased follow-up of patients after curative surgery for colorectal cancer. Even though more participants were treated with salvage surgery with curative intent in the intensive follow-up group, this was not associated with improved survival.Expert recommendations suggest that prostate cancer survivors should have annual digital rectal examinations, and that PSA levels should be monitored every 6 months for 5 years, and then annually (SOR C). Family physicians should be aware of survivorship care plans as tools to improve care coordination and outcomes for cancer survivors.

The National Institute on Alcohol Abuse and Alcoholism screening question for alcohol use disorders is administered to a 39-year-old male and is positive for drinking more than five drinks in one setting at least once in the last 3 months. Which one of the following would be the best follow-up measure to identify an alcohol use disorder in this patient? The CAGE questionnaire The Alcohol Use Disorders Identification Test (AUDIT) The Michigan Alcoholism Screening Test (MAST) Serum measurement of γ-glutamyl transferase An AST/ALT ratio

B Formal screening instruments such as the CAGE questionnaire, the Alcohol Use Disorders Identification Test (AUDIT), and the self-administered Michigan Alcoholism Screening Test (MAST) are the most effective method for screening for alcohol disorders in primary care. Although the CAGE questionnaire can identify patients with alcohol abuse and dependence, AUDIT is more effective for detecting hazardous or harmful drinking, with a sensitivity of 57%-97% and a specificity of 78%-96%, and is recommended by the U.S. Preventive Services Task Force. The MAST is 25 questions and may be more challenging to complete in a busy practice. The AUDIT is 10 questions and can be completed quite easily.The AUDIT includes questions about the quantity and frequency of alcohol use, as well as binge drinking, dependence symptoms, and alcohol-related problems. Its strength lies in its ability to identify people who may not be dependent. Research shows that the AUDIT may be useful when screening women and minorities. This screening tool also has shown promising results when tested in adolescents and young adults but it is less accurate in older patients.Biologic markers, such as aspartate aminotransferase, mean corpuscular volume, and γ-glutamyl transferase, do not work well as screening methods for alcohol problems in primary care.

One of your patients asks if her children should receive HPV vaccine. She has 12-year-old and 22-year-old daughters and a 16-year-old son. None of them have received HPV vaccine.Which one of the following would be appropriate advice about HPV vaccine? Only her daughters should receive the vaccine All of the siblings should receive the vaccine Her 13-year-old daughter should receive a three-dose series The vaccine is not recommended for patients who are already sexually active The vaccine is about 75% effective in reducing the HPV subtypes that cause HPV cancers

B HPV is the most common sexually transmitted infection in the world and there is good evidence that most individuals who are sexually active will be exposed to HPV at some time. It is estimated that 20 million people in the United States are infected with this virus.HPV vaccine is highly immunogenic and efficacious, and is very well tolerated. The CDC's Advisory Committee on Immunization Practices (ACIP) recommends routine administration of the HPV vaccine for both males and females at 11-12 years of age, so that they are vaccinated before becoming sexually active. The minimum age approved for the vaccine is 9 years. In addition, catch-up vaccination has been recommended for females ages 12-26 if they have not been previously vaccinated or if they have not completed the vaccine series (SOR A). The vaccine may also be given to those age 26-45 after a discussion of the benefits and shared decision-making, although the benefit is much lower in this age group. According to the ACIP, people in this age group who are not in a monogamous relationship and are sexually active should still be offered the vaccine, as well as those who are sexually active with a new partner.The recommended HPV series is two doses for those who start the vaccine series before the age of 15. The second dose should be given 6-12 months after the first dose. For patients who begin the vaccine series at age 15 or after, a three-dose schedule is recommended by the CDC, with the second dose given 1-2 months after the first dose and the third dose 6 months after the first dose.HPV types that cause most HPV cancers and genital warts have dropped 86% among teen girls. Among vaccinated women the percentage of cervical precancers caused by the HPV types most often linked to cervical cancer has dropped by 40%.

You are counseling a 45-year-old male with elevated LDL-cholesterol. When discussing dietary changes to promote healthy lipid levels, which one of the following would be accurate advice? He should minimize his consumption of nuts The Dietary Approaches to Stop Hypertension (DASH) diet recommended for reducing hypertension will help lower his LDL-cholesterol level Saturated fats should comprise 15% or less of his caloric intake He should aim for a fiber intake of 25 g daily He should record what he has eaten in a food diary at the end of each day

B In 2013, the American Heart Association (AHA) issued lifestyle management guidelines designed to reduce cardiovascular risk. For adult patients with elevated LDL-cholesterol levels the AHA advises following diet plans such as the Dietary Approaches to Stop Hypertension (DASH) diet, the AHA diet, or the USDA Food Pattern. The AHA specifically recommends reducing the percentage of calories from saturated fat, aiming for a goal of 6%-7% of calories from this source. The AHA also recommends a diet that emphasizes the consumption of fruits, vegetables, and whole grains, and which includes fish, poultry, low-fat dairy products, legumes, nontropical vegetable oils, and nuts. Consumption of red meat, sweets, and sugar-sweetened beverages should be discouraged.Although dietary fiber has been shown to have several beneficial health effects, the average daily intake for most Americans is 15 g daily, which is much lower than the recommended amount. The recommended daily fiber intake for males age 14-50 is 38 g daily. For other populations the recommended amount is lower, and varies according to age and sex. Several randomized, controlled trials have shown a reduction of LDL-cholesterol with higher fiber consumption. A food diary is an important aspect of dietary behavior change but it is most accurate if entries are made immediately after food is consumed.

You provide care for an extended family that lives in the same household and includes a 69-year-old female, her 44-year-old daughter, her 23-year-old granddaughter, and the granddaughter's 15-month-old infant. The family receives housing assistance and participates in the Supplemental Nutrition Assistance Program (SNAP). All of the family members are asymptomatic. The grandmother is a former smoker with a 30-pack-year smoking history. The 23-year-old is not married and her only medication is an oral contraceptive.Which one of the following would be recommended by the U.S. Preventive Services Task Force? Screening the 15-month-old for autism Screening the 23-year-old for gonorrhea Screening the 44-year-old for thyroid disease Screening the 69-year-old for abdominal aortic aneurysm

B The U.S. Preventive Services Task Force (USPSTF) recommends screening for both gonorrhea and Chlamydia in sexually active women 24 years of age and younger (B recommendation). The USPSTF has found insufficient evidence for screening nonpregnant asymptomatic adults for thyroid dysfunction (I recommendation). Although the USPSTF recommends abdominal aortic aneurysm (AAA) screening by ultrasonography in men 65-75 years of age who have ever smoked, it found that current evidence is insufficient to assess the balance of benefits and harms of screening for AAA in women 65-75 years of age who have ever smoked (I recommendation).It is important to screen all children for developmental delays, especially those who are at a higher risk for developmental problems due to preterm birth, low birth weight, or having a sibling or parent with autism spectrum disorder (ASD). The USPSTF has noted the need for more research around autism screening. However, the CDC recommends that all children be screened specifically for ASD with a validated tool during regular well child visits at 18 and 24 months of age. The American Academy of Pediatrics recommends screening at 18 months with a second screen at 24 months because of the increased likelihood of false-positives at 18 months. Universal screening at an earlier age is not as accurate with the current tools available in primary care.A number of tests are available to screen for ASD in children younger than 30 months. The most commonly studied tool is the Modified Checklist for Autism in Toddlers (M-CHAT). Additional specialized screening might be needed if a child is at high risk for ASD, such as children with a sibling with ASD, or if symptoms are present.

You are updating your office's immunization tracking system. According to the CDC, which one of the following patients would receive the most benefit from pneumococcal conjugate vaccine (PCV13, Prevnar 13)? A 45-year-old female who smokes cigarettes and has not previously received pneumococcal vaccine A 56-year-old male with a cochlear implant who received pneumococcal polysaccharide vaccine (PPSV23, Pneumovax 23) at age 54 A 65-year-old healthy male who has never received pneumococcal vaccine A 65-year-old female who has hypertension and underwent placement of a coronary artery stent 3 years ago and has never received pneumococcal vaccine A 76-year-old female with well controlled type 2 diabetes who received PPSV23 at age 65

B In June 2019 the CDC's Advisory Committee on Immunization Practices (ACIP) removed the recommendation for routine pneumococcal conjugate vaccine (PCV13) for adults ≥65 years of age and recommended administration of PCV13 based on shared clinical decision-making for adults ≥65 years of age who do not have an immunocompromising condition, cerebrospinal fluid (CSF) leak, or cochlear implant, and who have not previously received PCV13. Some adults ≥65 years of age are potentially at increased risk for exposure to PCV13 serotypes, however, and might gain a higher than average benefit from PCV13 vaccination. This includes people who live in nursing homes or other long-term care facilities, or in settings with low pediatric PCV13 uptake, and people who travel to areas with no pediatric PCV13 program.All adults ≥65 years of age should continue to receive one dose of pneumococcal polysaccharide vaccine (PPSV23). If the decision is made to administer PCV13, it should be given at least 1 year before PPSV23.ACIP continues to recommend PCV13 in series with PPSV23 for adults ≥19 years of age with an immunocompromising condition, CSF leak, or cochlear implant. Widespread use of 7-valent pneumococcal vaccine (PCV7) and PCV13 in children has led to sharp declines in pneumococcal disease among unvaccinated children and adults by preventing carriage, and thereby transmission, of strains covered by PCV13.

A 45-year-old male executive requests a "complete physical." He has been working out twice a week at a local health club. He does not smoke. His parents are alive and healthy, but his father takes a statin. There is no history of diabetes mellitus.On examination his BMI is 28 kg/m2 and his blood pressure is 132/84 mm Hg. He reports no change in his weight or diet over the last year. Laboratory findings from last year include a total fasting cholesterol level of 220 mg/dL, an HDL-cholesterol level of 38 mg/dL, an LDL-cholesterol level of 138 mg/dL, and a fasting glucose level of 108 mg/dL.Which one of the following would be most appropriate at this time? A repeat fasting lipid panel A hemoglobin A1c test A 2-hour glucose challenge test No laboratory testing

B Patients with a previous fasting glucose level >100 mg/dL should be retested for diabetes mellitus using either a validated hemoglobin A1c test or a repeat fasting glucose level. A 2-hour glucose challenge would require a separate visit and fasting status and would potentially place an additional burden on the patient to make an additional visit to the clinic and wait the 2 hours required for testing. A repeat lipid panel is not necessary for this patient, as his cardiac risk is only 3.5% based on the American College of Cardiology/American Heart Association risk stratification.

A 72-year-old female sees you for a routine evaluation. She is in good health but says that she recently slipped and fell while gardening and is now concerned that she may fall again and break her hip. She asks if there is anything she can do to prevent further falls.Which one of the following measures would be most effective for reducing the risk of falls in elderly patients? An environmental assessment of the home A vision assessment A stretching program Calcium supplementation, 1500 mg daily Vitamin D supplementation, 50,000 IU per week

B Statistics show that at least one-third of adults age 65 and older fall annually. Falls are the leading cause of nonfatal injuries and place the victims at increased risk for subsequent premature death. As the population ages, preventing this common cause of morbidity is of growing importance.Studies have found numerous modifiable risk factors related to falls. A combination of vision assessment and exercise was associated with a reduced incidence of falls. Tai chi in particular appears to reduce the risks of falling (level of evidence 1). Neither karate nor stretching has been shown to help seniors prevent falls.Environmental assessments by themselves have yielded mixed results in fall prevention in otherwise average-risk persons. Trials of different vitamin D formulations (with or without calcium), dosing schedules, and varying baseline fall risk show mixed results at 9-36 months of follow-up. A single trial of annual high-dose cholecalciferol (500,000 IU) showed an increase in fall risk. Increasing calcium intake by itself has not been shown to reduce the incidence of falls.

A 32-year-old female who is new to your practice sees you for a health maintenance visit. She does not take any medications and exercises regularly. She does not have a family history of coronary artery disease and does not recall having her cholesterol levels measured in the past. On examination her BMI is 24 kg/m2 and her blood pressure is 124/76 mm Hg.Which one of the following should you recommend regarding screening for this patient? Scheduling a fasting lipid profile for another day A lipid profile and counseling on diet and exercise now A lipid profile and a 10-year cardiac risk assessment now A lipid profile now and treatment with a high-dose statin if her LDL-cholesterol level is 170 mg/dL A lipid profile when she is 40 years of age

B Ten-year cardiac risk assessment calculators have not been validated for persons under the age of 40. However, since much of the process of atherosclerosis begins in young adulthood, the 2018 American Heart Association guidelines recommend screening for hyperlipidemia starting at age 20 with either a fasting or nonfasting lipid profile and a lifetime risk assessment (SOR B). If the serum triglyceride level is >400 mg/dL on a nonfasting specimen a fasting test is recommended. Patients under the age of 40 with severe primary hypercholesterolemia (LDL-cholesterol level ≥190 mg/dL) are considered to be at high risk for atherosclerotic cardiovascular disease and familial hypercholesterolemia, and maximally tolerated statin therapy is recommended. Assessing for risk-enhancing factors is also recommended. These include a family history of early coronary artery disease (before age 55 in males or 65 in females), metabolic syndrome, chronic kidney disease, chronic inflammatory diseases (systemic lupus erythematosus, psoriasis, and others), a history of preeclampsia, or South Asian ancestry. These factors should be taken into account if the patient's LDL-cholesterol level is 160-189 mg/dL. Statins are not currently recommended for patients with these risk factors who are under the age of 40, and more research is needed to define the benefits of statin treatment in this population.

The abbreviated Fagerström Test for Cigarette Dependence is used to determine the intensity of addiction in smokers. This test consists of which one of the following sets of questions? Do you have significant cravings in places where smoking is forbidden, such as church, the library, or movies? How soon after you wake up do you smoke your first cigarette? How soon after you wake up do you smoke your first cigarette? How many cigarettes do you smoke each day? Which cigarette would you most hate to give up? If you get a minor cold or illness do you continue to smoke the same number of cigarettes? What is the longest time you have gone without smoking? Do you smoke in your car?

B The Fagerström Test for Cigarette Dependence, formerly called the Fagerström Test for Nicotine Dependence, is available in both a long form and an abbreviated form. Use of this test can help a physician determine the intensity of a smoker's addiction, and thereby help determine dosages for medications used to help smokers quit.The short form of the test asks only when the patient smokes the first cigarette of the day and how many cigarettes are smoked each day. These questions have been shown to be both valid and reliable.The long form asks about smoking at inappropriate places and times, including during illnesses, and about when cravings are strongest.

A 62-year-old female sees you for a health maintenance visit. She does not take any medications and stays active by swimming three times a week. She asks about screening for osteoporosis since her mother was treated for osteoporosis but never had any fractures.Which one of the following is true regarding osteoporosis screening for this patient? The U.S. Preventive Services Task Force (USPSTF) recommends that routine bone measurement testing begin at age 60 for any woman who has a family history of osteoporosis, so this patient should be screened with bone measurement testing now The USPSTF recommends routinely screening all women for osteoporosis with bone measurement testing starting at age 65, so this patient should be screened in 3 years Once this patient undergoes bone measurement testing, if the results are normal she should be screened every 2-3 years The patient should be screened with bone measurement testing now if she has a FRAX score of 7.5%

B The U.S. Preventive Services Task Force (USPSTF) recommends screening for osteoporosis with bone measurement testing in women age 65 and older (B recommendation). In postmenopausal women younger than 65 the USPSTF recommends screening with bone measurement testing in those at increased risk for osteoporosis based on results from a formal risk assessment tool (B recommendation). Formal clinical risk assessment tools include the Simple Calculated Osteoporosis Risk Estimation (SCORE), the Osteoporosis Risk Assessment Instrument (ORAI), the Osteoporosis Index of Risk (OSIRIS), and the Osteoporosis Self-Assessment Tool (OST). The commonly used threshold to identify an increased risk for osteoporosis or osteoporotic fractures is 6 for SCORE, 9 for ORAI, <1 for OSIRIS, and <2 for OST.The FRAX tool uses the following factors to determine a risk percentage: Parent history of fracture Previous personal fracture Smoking status Glucocorticoid use Rheumatoid arthritis Secondary osteoporosis conditions Alcohol use Age, sex, weight, and height The FRAX tool was developed at the University of Sheffield and provides a country-specific, computerized algorithm that calculates the 10-year probability of hip fracture and major osteoporotic fracture. In women younger than 65 the USPSTF threshold for screening with bone measurement testing is a 10-year major osteoporotic fracture risk of 8.4%, which exceeds that of a 65-year-old white female without major risk factors.Some observational and modeling studies have suggested screening intervals based on age, baseline bone mineral density, and calculation of the estimated time until the patient develops osteoporosis. However, limited evidence from two good-quality studies found no benefit from repeating bone measurement testing 4-8 years after initial screening.

A 50-year-old female sees you for a routine health maintenance visit. She is asymptomatic and has no known family history of cancer. She underwent breast augmentation surgery 20 years ago. On examination she has a BMI of 22 kg/m2.According to the U.S. Preventive Services Task Force, which one of the following has the best evidence for breast cancer screening for this patient at this time? Monthly breast self-examinations Mammography only A clinical breast examination and mammography Breast MRI only Mammography, followed by breast MRI if dense breast tissue is noted on mammography

B The U.S. Preventive Services Task Force (USPSTF) recommends screening mammography every 1-2 years beginning at age 50 for all women (B recommendation). The USPSTF recommends against teaching breast self-examination (D recommendation) and has found insufficient evidence to recommend for or against clinical breast examinations to screen asymptomatic women (I recommendation). Clinical trials have not found that mortality is improved in women screened with a clinical breast examination when done in concert with mammography. The patient should be provided with this information and the physician should ask about her preferences with regard to the clinical breast examination.The American Cancer Society has somewhat different recommendations for breast cancer screening as noted below: Women age 40-44 may choose to start annual screening mammography Women age 45-54 should have annual mammography Women age 55 and older should switch to mammography every 2 years, or may choose to continue annual screening Screening should continue as long as a woman is in good health and is expected to live 10 more years or longer All women should be familiar with the known benefits, limitations, and potential harms linked to breast cancer screening Which recommendations to follow are a source of controversy in clinical practice and for patient education.The USPSTF has concluded that the evidence is insufficient to assess the benefits and harms of MRI screening for breast cancer (I recommendation). The American Cancer Society recommends annual MRI screening in addition to mammography for women with a ≥20% lifetime risk for breast cancer, and recommends against MRI screening for women with a ≤15% lifetime risk (SOR C). There is insufficient evidence at this time to make a recommendation for or against yearly MRI screening for women who have a higher lifetime risk based on certain factors such as a mammogram showing "extremely" or "heterogeneously" dense breasts. Some small studies have shown a benefit from MRI after mammography but guidelines do not currently recommend this.Studies looking at MRI-based cancer detection rates almost invariably look at the effectiveness of MRI after mammography because some cancers detected on mammography will not be seen on MRI. Cancer screening guidelines for women with augmented breasts are generally the same as for other women.The Breast Cancer Risk Assessment tool, also known as the Gail Model, is another tool clinicians can use to estimate a woman's risk of developing invasive breast cancer over the next 5 years, and her lifetime risk up to age 90. The tool uses a woman's personal medical and reproductive history and the history of breast cancer among her first degree relatives (mother, sisters, daughters) to estimate absolute breast cancer risk, defined as the probability of developing invasive breast cancer within a specific age interval.

Your clinic is planning for administration of this year's influenza vaccine. Which one of the following statements is true? Influenza vaccine should not be given until early October Influenza vaccine has a relatively low impact in children because cases tend to be milder and children are therefore not as infectious to others Patients with a history of urticaria after eating eggs do not need to be monitored for 30 minutes after administering influenza vaccine Influenza vaccine should never be given if there is a previous history of severe allergic reaction to eggs, such as hives or angioedema

C According to the CDC guidelines influenza vaccine should be administered before the end of October, but it can be given throughout the entire influenza season. It can also be given at regular health care visits before October if that is more convenient for the patient, and if there is a likelihood that the patient might not get the vaccine otherwise. Emphasis should be placed on vaccinating individuals prior to the start of influenza activity in the community (SOR A). When the vaccine is closely matched to the antigenic strains circulating in the population there are decreases in antibiotic use, hospitalization, absenteeism, and the use of health care resources in general (SOR B). Viral shedding studies within family units indicate that children with influenza shed the virus longer and have a greater potential to become infected, so vaccination of children for influenza is an important public health intervention to reduce community disease burden.

A 54-year-old male tells you that he has started an exercise program. His routine consists of chest presses, biceps curls, shoulder presses, abdominal crunches, and quadriceps extensions. He says he works out 3 days a week, completing two sets of 10 repetitions for each exercise.Which one of the following adjustments to his routine would you recommend? The frequency of training should be increased to at least five times per week He should strive for a target heart rate of 50%-60% of his maximum rate in the middle of his routine He should train the front and back of major muscle groups He should increase the number of repetitions to 20-30 in each set He should do fewer repetitions with heavier weights to maximum effort

C Although aerobic exercise has traditionally been emphasized for its health benefits, research increasingly suggests that complementary resistance training also has favorable effects on cardiovascular function, coronary risk factors, and physical and psychosocial well-being. The American Heart Association recommends the inclusion of resistance training for healthy persons of all ages, and for many patients with chronic diseases, including cardiovascular disease (SOR C). Programs that include a single set of 8-10 different exercises performed 2-3 days a week have been shown to be beneficial. Although a greater frequency of training is an option, the additional gain is usually small.While the number of exercises can be reduced, training the front and back of major muscle groups (e.g., chest/back, biceps/triceps) is recommended. A repetition range of 8-12 is recommended for healthy participants younger than 50-60 years of age. To reduce the risk for injury, 10-15 repetitions at a lower relative resistance is generally recommended for cardiac patients and healthy participants over 50-60 years of age. Higher-intensity efforts (fewer repetitions with heavier weights) increase the risk of musculoskeletal injury.The American College of Sports Medicine recommends that older adults perform the following each week: a minimum of 150 minutes of moderate-intensity aerobic activity or 75 minutes of vigorous-intensity aerobic activity, and two or more nonconsecutive days of moderate-intensity strengthening activities, with 8-10 exercises involving the major muscle groups and 8-12 repetitions of each exercise.

An 11-year-old female is brought to your office by her mother because of recurring headaches and abdominal pain. A detailed workup for her symptoms is negative thus far. She lives with her mother and her mother's boyfriend. When you ask permission and question the child alone in the room she tells you that her mother's boyfriend yells at her and her mother. The mother reports that she is always in the house when the boyfriend is there.Which one of the following is true in this situation? The information provided by the mother makes it very unlikely that sexual abuse has occurred The child's age makes it unlikely that sexual abuse has occurred The child's genital area should be examined if this was not already done You should question the boyfriend about possible abuse You should schedule a follow-up visit and reevaluate the child in 3-4 months

C Child abuse is common in the United States and should be considered when there is apparent neglect of a child, or indications of sexual, emotional, or physical abuse (SOR C). Specifically, child sexual abuse is defined by the Child Abuse Prevention and Treatment Act as "the employment, use, persuasion, inducement, enticement, or coercion of any child to engage in, or assist any other person to engage in, any sexually explicit conduct or simulation of such conduct for the purpose of producing a visual depiction of such conduct; or the rape, molestation, prostitution, or other form of sexual exploitation of children, or incest with children."Most cases of child sexual abuse are not reported, and less than 10% of child sexual abuse cases that are substantiated are associated with physical signs of abuse on examination. This figure highlights the great importance of careful history taking and a high index of suspicion if risk factors are present.Risk factors for child sexual abuse include female sex, parental alcoholism or substance abuse, intimate partner violence (domestic violence) in the home, and poor parental attachment. Children who have experienced one type of abuse are at risk for other types of abuse or neglect. Social isolation doubles the risk of abuse in girls. Other factors that place a child at increased risk of abuse or neglect include a history of prematurity, behavioral problems, medical fragility, and other special needs. Poverty and high local unemployment rates also increase the risk for child abuse. Preadolescent children between the ages of 10 and 12 are the most at-risk age group for child sexual abuse. There is a smaller peak of abuse in the 6- to 7-year-old age range.The presence of a caretaker in the home who has no biological relationship to the child increases the risk for abuse (sexual, emotional, and physical) as well as neglect. Most perpetrators are known to the child. Other caregiver risk factors for abuse and neglect include a past criminal history, a past history of mental health problems, misconceptions about child care and child development, and inappropriate expectations of the child. A caregiver with a substance abuse problem increases the risk for child sexual abuse and maltreatment.Because there was a negative evaluation for the child's headaches and abdominal pain, the possibility of sexual abuse should be considered and a genital examination is needed. If the family physician is not comfortable performing the examination then referral to an experienced clinician is necessary.

A 32-year-old male presents to your office for help with his drug abuse problem. He has abused methamphetamine for 8 years and was recently incarcerated for methamphetamine-related charges.Which one of the following is true regarding this situation? Symptoms related to methamphetamine withdrawal are similar to those seen with opioid and sedative withdrawal Buprenorphine and acamprosate have been approved by the FDA for managing withdrawal from methamphetamine or other stimulants Methamphetamine withdrawal is frequently associated with profound dysphoria, suicidal ideation, and suicide attempts Persistent headaches are expected during methamphetamine withdrawal and do not require further evaluation Pharmacologic therapy is the cornerstone of substance abuse treatment for methamphetamine addiction

C Cocaine and amphetamines (such as methamphetamine) are the most commonly abused stimulants. Detoxification involves interventions targeted at managing the acute intoxication, as well as the withdrawal period. It is the first step for patients who wish to become abstinent or who are in mandatory abstinence programs. In contrast, treatment or rehabilitation is the provision of ongoing services with the goal of promoting recovery.Family physicians are often called on to assist in detoxification in both outpatient and inpatient settings. In addition to detoxification the physician must address comorbid psychiatric illness and general medical disease. Comprehensive psychiatric management is the cornerstone of substance abuse treatment (SOR A) and will usually necessitate referral. Treatment programs should also integrate psychosocial aspects of care in a comprehensive treatment strategy.The symptoms of withdrawal from methamphetamine differ from those of alcohol, opioids, or sedatives. Common symptoms include fatigue, anxiety, irritability, depression, poor concentration, hypersomnia, psychomotor retardation, increased appetite, drug craving, and paranoia.Acamprosate is used for the management of alcohol dependence and is not indicated for stimulant addiction. Buprenorphine is approved for opioid dependence and has been used in some studies for stimulant addiction. In these studies it was more beneficial than methadone. There is limited evidence that some medications may be helpful in amphetamine dependence and abuse. An international study suggests using risperidone, aripiprazole, topiramate, and buprenorphine in certain situations. Haloperidol and risperidone reduced psychosis. Riluzole, a drug approved for treatment of amyotrophic lateral sclerosis (ALS), reduced craving, withdrawal, and depression compared with placebo. In a systematic review from 2020, the most consistent positive findings were demonstrated with stimulant agonist treatment (dexamphetamine and methylphenidate), naltrexone, and topiramate. Less consistent benefits have been shown with the antidepressants bupropion and mirtazapine.A commonly overlooked, and possibly lethal, component of stimulant withdrawal is profound dysphoria, involving negative thoughts and feelings and depressed mood. This may lead to suicidal ideation and attempts. The depressed affect and dysphoria associated with stimulant withdrawal are more profound and often longer lasting for patients who abuse methamphetamines. Thus, these patients warrant careful monitoring and treatment for depression and suicidality.Patients who are withdrawing from stimulants, including methamphetamines, often report headaches. However, persistent headaches may be due to intracerebral, subarachnoid, or subdural bleeding, and must be evaluated appropriately.

An obese 35-year-old female asks if you can prescribe "diet pills" for her. Her BMI is 32 kg/m2. She is otherwise healthy, but her mother and father both have diabetes mellitus and hypertension. An examination is notable only for a blood pressure of 138/86 mm Hg. Her hemoglobin A1c is 5.7%.Which one of the following is true? Starting a graded exercise program is more effective than using diet strategies to reduce weight Pharmacologic therapy almost always produces an average weight loss of >5 kg (11 lb) over 1 year Orlistat (Alli, Xenical) may reduce her risk for developing type 2 diabetes Hyperkalemia and metabolic alkalosis have been linked to the use of phentermine/topiramate (Qsymia) Pharmacologic therapy has been shown to reduce morbidity and mortality from obesity-related conditions

C Critique: A meta-analysis of 79 clinical trials involving diet plus sibutramine, orlistat, and phentermine, as well as several other drugs not approved for weight reduction, showed a placebo-adjusted overall average weight loss of <5 kg (11 lb) at 1 year. Long-term weight loss maintenance is very challenging using pharmacologic methods only, so there are no studies to demonstrate long-term morbidity and mortality benefits. Although they are not FDA approved for diabetes prevention, several medications have been shown to decrease incident diabetes mellitus. This includes several medications used for the treatment of diabetes, such as metformin, α-glucosidase inhibitors, GLP-1 inhibitors, and thiazolidinediones, as well as the weight loss drug orlistat.The relative risk of diarrhea and flatulence was >3.1 with orlistat compared to placebo. Since phentermine is a sympathomimetic amine, palpitations, tachycardia, and elevated blood pressure can be expected as side effects, but the drug is not contraindicated unless the patient has moderate to severe hypertension (SOR A). Topiramate inhibits carbonic anhydrase activity and has been linked to hypokalemia, kidney stone formation, and hyperchloremic, nonanion gap metabolic acidosis. For patients started on phentermine/topiramate a basic metabolic panel that includes bicarbonate, creatinine, potassium, and glucose is recommended at baseline and periodically during treatment. Exercise and diet changes with pharmacotherapy will produce the greatest changes in weight.

A 58-year-old male has increasingly symptomatic osteoarthritis of the knee. He says that acetaminophen no longer treats his knee pain effectively, but he has tried ibuprofen, 600 mg three times daily with food, and says that it works much better. He does not want to have surgery because his construction firm has a project with a deadline coming up in the next 3 months. He does not have hypertension or chronic kidney disease and does not take any other medications. He is up to date on all health prevention measures and when he was seen for a headache in the emergency department 3 months ago a metabolic profile was normal. He is hesitant to get an injection of his knee at this time. Which one of the following would be most appropriate at this point? Order a hemoglobin level so there is a baseline if he develops gastrointestinal bleeding Advise against ibuprofen and prescribe low-dose tramadol to preserve kidney function Order Helicobacter pylori testing and treat if positive, then recommend continuation of ibuprofen Recommend an over-the-counter or standard-dose prescription H2-blocker and continuation of ibuprofen

C Medical therapy for osteoarthritis should begin with full-strength acetaminophen and topical therapy, then proceed to NSAIDs and selectively to tramadol and other opioids. NSAIDs and opioids may reduce pain and improve function but have significant potential harms (SOR A). Based on meta-analyses of randomized, controlled trials physicians should perform laboratory screening for and eradicate Helicobacter pylori before initiating long-term NSAID therapy in NSAID-naive patients to reduce the risk of peptic ulcer disease (SOR A). Because of that finding the American College of Gastroenterology recommends testing for H. pylori infection before initiating long-term NSAID therapy, and offering eradication therapy to those with positive results (SOR A). Physicians should also screen for and eradicate H. pylori before initiating long-term NSAID therapy in patients with a history of peptic ulcers (SOR B). Treatment with standard dose H2-blockers is not effective for preventing peptic ulcers related to long-term NSAID use.Low-dose tramadol would not be recommended for this patient if other nonopioid medications provide relief. Establishing a baseline hemoglobin level is not necessary before starting NSAID treatment.

A 54-year-old male asks for a prescription for zolpidem (Ambien) to help him sleep. He has had trouble falling asleep and staying asleep for the last 4 weeks. He believes he is getting about 6 hours of sleep per night and feels tired towards the end of his workday. He does not feel depressed and has no new stresses at home or work.When counseling the patient about sleep, appropriate advice would include which one of the following? A majority of adults can function adequately on 6-7 hours of sleep a night Exercise in the evening before bed will make him sleepier Inadequate sleep is associated with the development of hypertension, diabetes mellitus, and obesity If he wakes up during the night and can't get back to sleep, watching television in another room for 30 minutes or so is recommended Sleeping 2 hours longer in the morning on days off is helpful if he needs to catch up on sleep

C On average most adults need 7-9 hours of sleep each night, although the normal range is 5-10 hours. Teenagers need an average of 9 hours of sleep each night, and infants need about 16 hours a day (SOR B). Like their younger counterparts, the elderly need 7-9 hours of sleep each night. However, sleep disorders increase with age, and the elderly are much more likely to have sleep problems, including insomnia and a lack of deep sleep (SOR C).Sleeping less than 7 hours per night on a regular basis is associated with adverse health outcomes, including weight gain and obesity, diabetes mellitus, hypertension, heart disease and stroke, depression, and an increased risk of death. Other effects include impaired immune function, increased pain, impaired performance, increased errors, and a greater risk of accidents.Exercise is important for sleeping well. However, patients should be advised to exercise several hours before going to bed to avoid the stimulating effect of exercise. Other recommendations include maintaining a regular sleep schedule including bedtime and wake-up time, avoiding caffeine and alcohol intake 4-6 hours before bedtime, and performing relaxing activities before bed (SOR C).Watching television and using computers or cell phones before bedtime is not recommended because exposure to bright light sources can interfere with sleep initiation. Use of these devices is particularly not recommended in the bedroom.

A 42-year-old female sees you for a routine health maintenance visit. Her neighbor was just diagnosed with ovarian cancer and has encouraged her to have her CA-125 level checked. The patient asks about ovarian cancer risk factors, prevention, and screening. Which one of the following would be appropriate advice? A past history of oral contraceptive use increases the risk for ovarian cancer Hormone replacement therapy after menopause decreases the risk for subsequent ovarian cancer CA-125 has a false-positive rate of 98% when used to screen for ovarian cancer Bimanual examinations are recommended to screen for ovarian cancer Transvaginal ultrasonography is recommended to screen for ovarian cancer

C Ovarian cancer is the fifth leading cause of cancer death among women in the United States. Risk factors associated with ovarian cancer include a positive family history and having the BRCA1 or BRCA2 gene mutation. A first or second degree relative with ovarian cancer increases the risk by about threefold. The use of oral contraceptives during the reproductive years, and pregnancy, especially after age 35, reduce the risk of ovarian cancer, but postmenopausal estrogen use may increase the risk.The U.S. Preventive Services Task Force does not currently recommend screening for ovarian cancer, as it is likely to have a relatively low yield (D recommendation). Almost all women with a positive screening test for CA-125 will not have ovarian cancer. In women at average risk, the positive predictive value of an abnormal CA-125 is approximately 2%, so 98% of women with positive test results will not have ovarian cancer. There are no current recommendations for ovarian cancer screening by either transvaginal ultrasonography or pelvic examination.

Which one of the following is an indication to offer abdominal ultrasonography to screen for an abdominal aortic aneurysm (AAA)? Hypertension and type 1 diabetes in a 60-year-old male Hypertension and a 20-pack-year smoking history in a 65-year-old female who quit smoking 4 years ago A 5-pack-year smoking history in a 68-year-old male who quit smoking 40 years ago No recent AAA screening in a 74-year-old male whose last screening ultrasonography 8 years ago was negative A recent history of hemorrhagic stroke in a 75-year-old female

C Smoking history (at least 100 cigarettes in a lifetime) and male sex are the major risk factors for abdominal aortic aneurysm (AAA). The U.S. Preventive Services Task Force (USPSTF) recommends one-time screening for AAA by ultrasonography between the ages of 65 and 75 in men who have ever smoked (B recommendation). The USPSTF also recommends that clinicians selectively offer screening for AAA in men in this age group who have never smoked if indicated by the patient's medical history, family history, other risk factors, or personal values (C recommendation). An important risk factor in addition to age is a first degree relative with AAA. Other risk factors include a history of other vascular aneurysms, coronary artery disease, cerebrovascular disease, atherosclerosis, hypercholesterolemia, obesity, or hypertension.The likelihood of finding an AAA large enough to benefit from surgery is greatest between the ages of 65 and 75. In patients older than 75 the likelihood of surviving surgery to repair an AAA is low enough to preclude screening. The benefit of screening for women in this age group is low due to the low number of AAA-related deaths in this population (SOR B). The USPSTF recommends against routine screening for AAA in women (D recommendation). The USPSTF does not make any recommendation regarding testing those less than 65 years old.

You are implementing a program to routinely screen your patients for substance abuse. Excluding tobacco and alcohol use, approximately what percentage of patients age 12 and older seen in a family practice setting have substance abuse problems? 3% 5% 10% 20%

C Substance abuse is usually defined as problematic use of alcohol, illicit drugs, or tobacco. Substance abuse frequently goes unrecognized in the primary care setting, making recognition of its prevalence critical.An estimated 10% of persons age 12 or older in the United States have an illicit drug problem. The number of people who abuse alcohol is even higher, with 24.9% of those in this age group classified as binge drinkers and 6.5% classified as heavy alcohol users. Patients with substance abuse problems are more likely to develop medical problems and more likely to access care frequently, compared to the general population.Typical red flags in the history of patients with substance abuse problems include relationship difficulties, unexplained trauma, DUI, and an erratic occupational history. In addition, patients with mental health disorders have a higher prevalence of substance abuse, especially those with depression and personality disorders.The 2017 National Survey on Drug Use and Health provided a snapshot of the ongoing opioid epidemic and mental health across the country. The report found that 1 in 12 American adults (18.7 million) had a substance use disorder and that 1 in 5 (46.6 million) had a mental illness. More than 8.5 million were found to have both a substance use disorder and mental illness. In 2018 an update of the survey found that nearly 22 million people in the United States had a substance use disorder, with nearly 58 million people in the United States having either a substance use disorder or a mental health disorder.The National Institute on Drug Abuse website includes useful research, resources for clinical practice, and toolboxes (http://www.nida.nih.gov).The references cited also provide a wealth of information on substance use for many different drugs and segments of the population and should be utilized to obtain the most up-to-date statistics.

A 58-year-old female with a history of epilepsy wants to stop smoking due to a recent diagnosis of coronary artery disease. She has not taken any medication for seizures for the last 5 years and has not had a seizure during that time. She has been asking friends about what they have used to stop smoking, and she asks specifically about using bupropion (Wellbutrin SR, Zyban), varenicline (Chantix), nicotine replacement therapy, or other treatments to help her stop smoking.Which one of the following would you tell her? Bupropion should not be combined with varenicline for smoking cessation Bupropion should not be combined with nicotine replacement therapy Bupropion should be avoided because of her coronary artery disease Bupropion should be avoided because of her history of seizures

D Bupropion is an atypical antidepressant which is effective for tobacco cessation and is thought to inhibit dopamine and norepinephrine reuptake. Bupropion sustained-release formulations are approved for smoking cessation. A Cochrane review of antidepressants for smoking cessation found that both bupropion and nortriptyline are effective, while other tricyclic antidepressants, SSRIs, and anxiolytics are ineffective.Adding bupropion to varenicline does not lead to increased rates of smoking cessation. There is no evidence that bupropion or varenicline increases the risk of coronary artery disease (SOR A).The 2008 Public Health Service tobacco cessation guideline revision confirmed the utility of combining bupropion with nicotine replacement therapy (SOR A). Bupropion is combined with either the nicotine patch or an intermittent-dose formulation of nicotine replacement, or with both the patch and an intermittent dose formulation for patients with significant breakthrough cravings.Bupropion is contraindicated in patients with a present or past history of seizure disorders, a previous history of significant head injury, or anorexia nervosa or bulimia, as well as those taking another medication that lowers the seizure threshold.

Which one of the following is true regarding screening for drug abuse? The benefits of screening adolescents for drug abuse are clear Counseling adolescents and young adults about drug abuse has been shown to prevent them from abusing drugs Screening is most effective when done in the context of a preventive services visit The U.S. Preventive Services Task Force recommends screening all adults for unhealthy drug use

D Drug use is one of the most common causes of preventable injuries, disability, and death. Data from 2018 showed that an estimated 12% of U.S. residents 18 years or older reported current unhealthy drug use. Unhealthy drug use was reported by 24% of adults age 18-25, 10% of older adults, and 8% of adolescents age 12-17. The U.S. Preventive Services Task Force (USPSTF) now recommends screening by asking questions about unhealthy drug use in adults age 18 years or older. Screening with written or verbal questions should be implemented when services for accurate diagnosis, effective treatment, and appropriate care can be offered in the practice or referred. The USPSTF does not recommend drug testing as a form of screening.The USPSTF states that there is currently insufficient evidence to assess the balance of benefits and harms of broad-based screening of adolescents. The data on counseling adolescents on drug use is mixed. Some smaller trials show some benefit from brief intervention for low-risk drug use such as cannabis and alcohol. Other studies fail to demonstrate a consistent benefit and can be difficult to interpret due to the complexity of meta-analyses and comparisons.The USPSTF does not make any recommendations on the timing or setting of screening. Screening is appropriate at any visit at clinics where care or referral for unhealthy drug use can occur. Practices may consider brief tools such as the National Institute on Drug Abuse (NIDA) Quick Screen, which asks four questions about use of alcohol, tobacco, and illegal drugs, along with nonmedical use of prescription drugs, in the past year. Longer tools like the eight-item Alcohol, Smoking, and Substance Involvement Screening Test (ASSIST), which assess risks associated with unhealthy drug use or comorbid conditions, may reveal information signaling the need for prompt diagnostic assessment. Another assessment is the Tobacco, Alcohol, Prescription medication, and other Substance use (TAPS) tool, which may be useful when clinicians are concerned about misuse of prescription medications.

A 55-year-old male expresses concern about his inability to maintain an erection that allows for satisfactory sexual intercourse with his wife. He takes over-the-counter diphenhydramine (Benadryl) at night for sleep and takes a daily multivitamin. He says he drinks one 12-ounce beer 2-3 times per week. A physical examination is normal, including his blood pressure.Which one of the following would you tell him? Most cases of erectile dysfunction (ED) have a psychogenic etiology Diphenhydramine has little impact on his ED Abstaining from alcohol use will improve his symptoms Erectile dysfunction may be an early indication of vascular disease About 5% of men his age experience ED

D Erectile dysfunction (ED) is common, affecting an estimated 30 million men in the United States, and becomes more common with advancing age. The Health Professionals Follow-up Study reported moderate to severe ED in 12% of men younger than 59, 22% of men ages 60-69, and 30% of men older than 69.It was previously thought that the majority of cases of ED were caused by psychogenic factors such as family or occupational stress. However, evidence suggests that approximately 80% of ED is due to organic disease, which can be divided into hormonal, vasculogenic, and neurogenic causes. Vasculogenic etiologies are the most common, with arterial or "inflow" disorders accounting for more problems than venous disorders. The patient should be advised that their ED is a risk factor for underlying cardiovascular disease and that further evaluation may be appropriate. It is important to remember, however, that even though the primary etiology of ED is most often organic, psychological factors frequently coexist and play a role in the dysfunction.Many medications can cause or contribute to ED. It is estimated that as many as 25% of ED cases are due to medication side effects. This highlights the crucial role of the primary care physician in reviewing medication lists and modifying treatment regimens as part of addressing ED. Common offenders include antihistamines, antihypertensives and diuretics such as hydrochlorothiazide and spironolactone, psychoactive medications including SSRIs, and anti-epilepsy medications. It is not clear whether low amounts of alcohol cause erectile dysfunction.

You are counseling a 62-year-old male about colon cancer screening and he asks you about fecal DNA testing as an option. You find an article that says that the first-generation fecal DNA test has a sensitivity of 20% and a specificity of 96% for "screen-relevant" neoplasms. You are interested in the likelihood ratio of a positive test in this scenario.Which one of the following is the positive likelihood ratio of this test? 0.76 0.833 1.2 5.0

D Likelihood ratios indicate the probability that a patient has a disease with a positive test. Unlike positive and negative predictive values, likelihood ratios do not change with the prevalence of the disease in a given population. A likelihood ratio of 1 indicates that a positive test does not change the likelihood of the disease, and the higher the likelihood ratio is above 1, the more likely it is that a test rules in a disease. The positive likelihood ratio is equal to the sensitivity of the test divided by one minus the specificity. In the example given in this question, dividing 0.20 by 1.0 - 0.96 yields a positive likelihood ratio of 5.0. The negative likelihood ratio is calculated by one minus the sensitivity divided by the specificity.

A 20-year-old male is brought to your office by friends a few hours after they went dancing at a nightclub. His friends report that he has been combative and confused, and that he keeps clenching his jaw. Examination reveals a temperature of 38.2°C (100.8°F), a blood pressure of 160/94 mm Hg, and a heart rate of 108 beats/min. He has a mildly ataxic gait.Which one of the following club drugs is the most likely cause of these findings? Flunitrazepam (Rohypnol) GHB (γ-hydroxybutyrate) Ketamine MDMA (3,4-methylenedioxymethamphetamine)

D MDMA, flunitrazepam, GHB, and ketamine are among the drugs used by teens and young adults at nightclubs, bars, raves, or trance parties. Raves and trance parties are generally nightlong dances, often held in warehouses. Many who attend these dances use club drugs in an effort to enhance the experience.MDMA is a synthetic psychoactive drug that is chemically similar to the stimulant methamphetamine and the hallucinogen mescaline. Street names for MDMA include ecstasy, XTC, and hug drug. It is taken as a pill. Results from the 2018 National Survey on Drug Use and Health showed that 7% of people over the age of 12 had used MDMA at some time during their life and 1% had used it in the past year.MDMA users might feel very alert or energetic at first. At raves they can dance for hours at a time. They may also experience distortions in time and other changes in perception. Some, however, can become anxious and agitated. Sweating or chills may occur, and MDMA users may feel faint or dizzy. MDMA can interfere with the body's temperature regulation, which can cause dangerous hyperthermia. Other effects on the body include muscle tension, clenching of teeth, nausea, blurred vision, fainting, and chills or sweating. MDMA increases heart rate and blood pressure and can cause confusion, depression, sleep problems, intense fear, and anxiety. In regular abusers some of these side effects can last for days or weeks after taking MDMA.GHB, flunitrazepam, and ketamine have been referred to as date rape drugs, since they have been used to facilitate sexual assault. These drugs can be easily added to flavored drinks without the victim's knowledge. Symptoms of GHB intoxication include relaxation, drowsiness, vision problems, nausea/vomiting, headache, loss of consciousness, loss of reflexes, seizures, coma, and death. Flunitrazepam ("roofies") can cause sleepiness, a sensation of intoxication, visual and gastrointestinal disturbances, urinary retention, and loss of memory about the timespan when the person was under the drug's effects. Ketamine, which can also be snorted or smoked, is associated with a loss of time and identity, feeling out of control, a dream-like sensation, numbness, and increased heart rate and blood pressure.

A 15-year-old female sees you for a sports preparticipation evaluation. She has no known medical problems. There are no significant findings on her personal history or family history.Important components of the preparticipation evaluation for this patient include all of the following EXCEPT cardiac auscultation with the patient both standing and supine evaluation of femoral pulses vision and hearing screenings a urinalysis

D Obtaining a urinalysis is not recommended in asymptomatic individuals unless clinical signs such as elevated blood pressure warrant further investigation. Cardiac auscultation to detect murmurs is important to assess for any change in a heart murmur with a Valsalva type maneuver, which could indicate hypertrophic cardiomyopathy. Evaluation of femoral pulses can detect coarctation of the aorta. It is appropriate to perform hearing and vision screenings during a preparticipation evaluation.

The mother of 2-year-old twins, a boy and a girl, asks for advice to promote physical activity and help them maintain a healthy weight, especially as they get older. Which one of the following would be appropriate advice? Watching television for up to 2 hours per day does not increase the risk of physical inactivity Among adolescents, the levels of fitness and exercise over time are similar for males and females As children get older, programs to increase activity will generally be equally effective for both genders The physical activity level of children in relation to their peers is generally established by 3 years of age

D Physical activity levels of children are important to health and for avoiding obesity across the lifespan. While it may not be possible to determine to what extent activity levels are taught rather than inherent, it is clear that physical activity of children in relation to their peers is largely determined by 3 years of age.There is evidence that in children 8-10 years of age, television watching and activity levels are inversely correlated, as are activity levels and obesity. When children are induced to significantly reduce television watching, even below the recommended maximum of 2 hours per day, activity increases and BMI decreases.In adolescence, both boys and girls tend to reduce physical activity levels, but this is much more pronounced in girls. Girls may therefore need additional support and encouragement to maintain health-enhancing physical activity.

A 42-year-old male comes to your clinic for a health maintenance visit. He would like to start an exercise program. He is a nonsmoker and does not take any routine medications. His BMI is 27 kg/m2, his blood pressure is 142/92 mm Hg, and his hemoglobin A1c is 5.4%. He has a total cholesterol level of 230 mg/dL, an LDL-cholesterol level of 160 mg/dL, and an HDL-cholesterol level of 45 mg/dL.Which one of the following would be appropriate advice at this time? Exercise alone will not have much of an effect on his blood pressure He should start a statin medication He must lose weight in order to reduce his blood pressure Attenuation of potential atherosclerosis can occur with exercise even if he does not take a statin medication

D Regular aerobic exercise has been shown to reduce blood pressure in both normotensive and hypertensive individuals. Studies have shown a lowering of blood pressure even in the absence of weight loss. Regular aerobic exercise has been shown to reduce insulin resistance and, in conjunction with weight loss, has been shown to reduce the progression from prediabetes to type 2 diabetes. This patient is currently only slightly overweight and will not benefit significantly from weight loss alone.This patient's cardiac risk score is 4.5%, which is below the 7.5% threshold recommended by the American College of Cardiology/American Heart Association guidelines for statin therapy. (A heart risk calculator can be found at http://www.cvriskcalculator.com/.) The U.S. Preventive Services Task Force, however, recommends that adults with no history of cardiovascular disease (CVD) use a low- to moderate-dose statin for the prevention of CVD events and mortality when they are age 40-75; have one or more CVD risk factors, such as dyslipidemia, diabetes mellitus, hypertension, or smoking; and have a calculated 10-year risk of a cardiovascular event of 10% or greater. Because of this patient's cardiac risk score, he does not meet all three criteria.

A 45-year-old female sees you for counseling on smoking cessation. You tell her that stopping smoking reduces the risk of lung cancer almost immediately reduces the risk of laryngeal cancer within 5 years reduces the risk of bladder cancer within 5 years reduces the risk of certain types of leukemia

D The 2004 U.S. Surgeon General's Report expanded the list of smoking-related cancers, noting that the evidence either suggested or was strong enough to infer causal relationships between smoking and cancers of the lung, larynx, oral cavity, pharynx, esophagus, stomach, liver, pancreas, colon and rectum, uterine cervix, kidney, and bladder, as well as other sites. Smoking was also causally linked to adult myeloid leukemia. In 2014 the Surgeon General released a report marking 50 years since the first Surgeon General's report on smoking. This report also examined links between smoking and cancer, and discussed causal links to lung cancer, liver cancer, and colorectal cancer, as well as evidence suggesting a link between breast cancer and smoking. The 2014 report also notes that smoking does not cause prostate cancer. The evidence did show that smoking increases the risk of dying in patients with cancer, including those with breast or prostate cancer.The risk of cancer from smoking shows a clear relationship to both the number of cigarettes smoked each day and the duration of smoking; the earlier one begins to smoke, the higher the risk. While the increased risk of lung cancer due to smoking starts to decline within about 5 years among persons who stop smoking, the residual risk may persist for several decades. Stopping smoking before middle age greatly decreases the risk of lung cancer, and cessation sharply reduces the risk of laryngeal cancer within 10 years. The risk of bladder cancer, on the other hand, persists much longer after cessation (level of evidence 2 for all findings).

The American Diabetes Association recommends screening for diabetes mellitus and prediabetes in which one of the following patients? A 24-year-old female with a BMI of 32 kg/m2 who delivered a 3900-g (8 lb 10 oz) infant 18 months ago after having normal results on a 50-g glucose challenge test A 27-year-old female with polycystic ovary disease who has a BMI of 36 kg/m2 and had an HbA1c of 5.3% 12 months ago A 48-year-old white male who takes no medications, has no family history of diabetes, has a BMI of 34 kg/m2, and had an HbA1c of 5.3% 15 months ago A healthy 60-year-old white male who exercises regularly and had a normal fasting glucose level 4 years ago

D The American Diabetes Association (ADA) recommends screening all people over the age of 45 for diabetes mellitus every 3 years with a fasting plasma glucose level, a 2-hour oral glucose tolerance test, or a hemoglobin A1c (HbA1c). It is important to remember when using HbA1c for screening that hemolytic anemias and acute blood loss can falsely lower HbA1c, whereas prior splenectomy and aplastic anemias, which increase erythrocyte age, can falsely elevate HbA1c. Hemoglobinopathies and hemoglobin variants can result in variable changes in HbA1c level and may be more prevalent among certain racial and ethnic groups. When used for diagnostic purposes the HbA1c test should be performed using a method that is certified by the NGSP (http://www.ngsp.org) and standardized or traceable to the Diabetes Control and Complications Trial (DCCT) reference assay.Screening should be performed before age 45 for any individual with a BMI ≥25 kg/m2 (≥23 kg/m2 in Asian-Americans) who has any of the following additional risk factors: physical inactivity a low HDL-cholesterol level (<35 mg/dL) or a high triglyceride level (>250 mg/dL) a first degree relative with diabetes mellitus polycystic ovary syndrome or other insulin-resistance conditions such as acanthosis nigricans high-risk ethnicity, including African-American, Latino, Native American, Asian-American, and Pacific Islander a previous glucose tolerance test with elevated results or a hemoglobin A1c >5.7% a history of cardiovascular disease a blood pressure ≥140/90 mm Hg or receiving treatment for hypertension The ADA also recommends that women who have been diagnosed with gestational diabetes should receive lifelong screening every 3 years.U.S. Preventive Services Task Force (USPSTF) recommendations are slightly different. They recommend screening all adults 40-70 years of age who are overweight or obese (B recommendation). The USPSTF recommends that earlier screening or using a lower BMI cutoff should be considered for some patients, including those who have a family history of diabetes, have a history of gestational diabetes or polycystic ovary syndrome, or are members of certain racial/ethnic groups (African-American, Native American or Alaskan Native, Asian-American, Hispanic, or Native Hawaiian or Pacific Islander). If screening results are normal, repeat testing is recommended at a minimum of 3-year intervals.

You see a 45-year-old male who has smoked cigarettes for 25 years. He is very interested in quitting but has not been able to do so despite many attempts. He is interested in using medications to help.Which one of the following would likely be most effective? Calling the QUIT LINE and using over-the-counter nicotine patches Bupropion (Wellbutrin SR, Zyban) Nortriptyline (Pamelor) Varenicline (Chantix) Varenicline plus nicotine replacement therapy

E Not only are tobacco cessation treatments effective clinically, they are also cost-effective in comparison to treatments for other medical disorders (SOR A). Several analyses have found that the cost of treatment per patient who quits ranges from several hundred to a few thousand dollars. Insurance coverage of medications and counseling to stop smoking increases success rates (SOR A). Bupropion, varenicline, and five forms of nicotine replacement (gum, inhaler, lozenge, nasal spray, and patch) have all been shown to be effective in helping adults quit smoking (SOR A).For every 10 smokers who quit while taking a placebo nearly 30 could be expected to quit when taking varenicline as a single agent. Varenicline as a single agent has also been shown to help about 50% more people quit smoking compared to nicotine replacement therapy (NRT). Varenicline has been shown to be more effective than the nicotine patch (odds ratio [OR] 1.510), nicotine gum (OR 1.72), and other forms of NRT including inhalers, sprays, tablets, or lozenges (OR 1.42). However, varenicline was not shown to be more effective than combination NRT (OR 1.06). Combination NRT using a nicotine patch plus an additional form such as a lozenge also outperformed single NRT. A systematic review demonstrated that a combination of NRT and varenicline appears to have the highest quit rates.A meta-analysis of the bupropion and varenicline trials found no difference between the active drugs and placebo arms (risk ratio 1.06) with regard to neuropsychiatric events. Nortriptyline nearly doubles the chances of quitting but may have more side effects such as dry mouth. Unlike varenicline, neither nortriptyline nor bupropion was shown to enhance the effect of NRT compared with NRT alone.Telephone quit lines are also effective for tobacco cessation (SOR A). They reach a diverse population, and family physicians and other clinicians are encouraged to promote their use as supportive therapy when using pharmacologic approaches.

A 27-year-old patient asks about pharmacologic options to consider as part of her plan to lose weight. Accurate advice would include which one of the following? Weight loss is usually sustained after discontinuation of the agent Phentermine/topiramate (Qsymia) has the lowest success rates among pharmacologic agents Orlistat (Alli, Xenical) has the fewest side effects among the currently available agents The average weight loss using pharmacotherapy is 10 kg (22 lb) High-dose liraglutide (Saxenda) can be used for weight loss in patients who do not have diabetes

E Pharmacologic agents can be an effective part of a weight loss plan. However, patients should be counseled regarding realistic expectations and possible risks. The average weight loss is approximately 5 kg (11 lb) for many of the agents and much of the weight that is lost while taking these agents is regained after the drug is stopped (SOR A). Use of these agents in the absence of lifestyle interventions can actually decrease a person's ability to lose weight in the future (SOR B). In a meta-analysis liraglutide was associated with 63% of patients losing at least 5 kg. Phentermine/topiramate has the highest success rate, with 75% of patients losing at least 5 kg. Orlistat may be associated with a high incidence of side effects, especially diarrhea associated with consumption of a high-fat meal. Liraglutide may be used for weight loss in patients without diabetes. Even high dosages of up to 3 mg daily can be prescribed safely if the patient can tolerate the dose.

A 47-year-old female would like to lose weight and asks about the effectiveness of various diets. Which one of the following would be accurate advice? An average caloric deficit of 300 kcal daily will result in a weight loss of 1 lb per week Very-low-calorie diets (500 kcal daily) are more effective than low-calorie diets (800-1500 kcal daily) for weight loss sustained over a period of 1 year Low-carbohydrate diets such as paleo diets have a high long-term success rate Low-fat diets are superior to low-calorie diets for weight loss Intermittent fasting has been shown to produce weight loss similar to that seen with pharmacologic treatment

E To lose weight, a person must create a caloric deficit either by increased activity or decreased caloric intake. A caloric deficit of at least 3500 kcal should theoretically produce a weight loss of 1 lb, so a caloric deficit of 500 kcal daily is required to produce a 1-lb weight loss in 1 week (SOR A). There are numerous diets published, including low-calorie, very-low-calorie, low-fat, very-low-fat, and low-carbohydrate among others, but long-term compliance is problematic. Typically, one-third to one-half of weight loss is not maintained by diet alone.In one study, Paleo diets resulted in less weight loss compared with intermittent fasting and a Mediterranean diet. Very-low-calorie diets (400-500 kcal daily) may increase rates of early weight loss, but weight loss at 1 year is similar to that from a low-calorie diet (800-1500 kcal daily) (SOR A). Low-fat diets, with fat accounting for 10%-19% of calories, do not produce weight loss without a decrease in total caloric intake (SOR A). The Mediterranean diet and behavior modification, such as the use of meal-replacement shakes, has been proven to improve long-term weight loss. Intermittent fasting may produce results similar to pharmacologic means, resulting in a loss of about 9 lb.


संबंधित स्टडी सेट्स

Data Structures - Self-Review Questions 3

View Set

标准教程 HSK1 第一课 你好

View Set

advantages and disadvantage sole proprietorship

View Set

EXAM FX: Life and Health Stimulate Exam

View Set

Module 5: Government & the Economy

View Set

PSYCHOLOGY: Approaches- introspection

View Set

Pharm Chapter 3 Drug Action Across the Life Span

View Set

Radiographic Procedures III Final Review

View Set

Personal Finance Midterm Questions

View Set